LSAT and Law School Admissions Forum

Get expert LSAT preparation and law school admissions advice from PowerScore Test Preparation.

 anabil@umich.edu
  • Posts: 7
  • Joined: Apr 30, 2016
|
#23705
Hello,

I'm having an issue understanding why the answers to the question on page 249 of the logic reasoning book is not "A".

The conclusion states that "So if a nation wants to maintain its value system and way of life, it must not allow its highest tax bracket to exceed 30 percent of income."

The question stem is, "Each of the following, if true, weakens the politican's argument EXCEPT."
This means that we are looking for an answer that either strengths or does not affect the politicin's argument.

A) The top level of taxation must reach 45 percent before taxation begins to deter invesntors and industrialists from introducing new technologies and industries.

My reasoning is that this answer supports the conclusion and is therefore the correct answer because the politician argues that taxes should not exceed 30 percent of income. The answer states that at 45 percent will taxes deter investors and industrialists. 30 percent is less than 45 and therefore he is technically correct. At a maximum of 30 percent tax (which is less than 45), we will not deter investors.

Any help would be much appreciated.
User avatar
 Dave Killoran
PowerScore Staff
  • PowerScore Staff
  • Posts: 5850
  • Joined: Mar 25, 2011
|
#23751
Hi A,

Thanks for the question! I usually answer most of the LSAT Bibles questions, but I'm out of town at the moment and otherwise occupied. I will try to have one of our course developers answers this asap for you, however. My apologies for the delay.

Thanks!
 Nikki Siclunov
PowerScore Staff
  • PowerScore Staff
  • Posts: 1362
  • Joined: Aug 02, 2011
|
#23752
Hi A,

Thanks for your question. The author concludes that the highest tax bracket should not exceed 30 percent of income, but the specificity of this recommendation is not properly supported by the rest of the argument. We learn that nations with high income taxes will suffer politically and economically, but there is no reason why the highest tax bracket should be capped at exactly 30%. The author is clearly assuming that any income tax above 30% would be unreasonably high, triggering the chain of events outlined earlier in the argument.

His assumption is unwarranted. What if the negative consequences of income taxation do not become apparent until the highest tax bracket reaches 45% of income? If so, then we'd have little reason to heed the warning in the author's conclusion, as 30% would not be a level of taxation that would necessarily deter investors. Answer choice (A) therefore weakens the argument, making it an incorrect response to this Weaken-EXCEPT question.

Let me know if this makes sense.

Thanks,
 anabil@umich.edu
  • Posts: 7
  • Joined: Apr 30, 2016
|
#23885
Thank you for your explanation.

Get the most out of your LSAT Prep Plus subscription.

Analyze and track your performance with our Testing and Analytics Package.